アクチュアリー採用問題の解答案(5)

http://d.hatena.ne.jp/actuary_math/20100114
から始まった
アクチュアリー採用問題の解答案」
シリーズの5回目です。

http://d.hatena.ne.jp/actuary_math/20100129#c1264833219
でリクエストのあった
大同生命
http://d.hatena.ne.jp/actuary2/20090411/1239419156
を取り上げます。


この会社は今回取り上げる会社の中では一番数学科で履修する内容に近いものを出題していると考えられます。(なお、念のために申し上げますが、問題自体の難しさとは別の話ですのでご注意ください。)


問題1は、高校程度の問題ですが、場合分けが意外と面倒です。
問題4の特に(3)はいわゆるε−δ論法を使わないと解けない問題となっています。
また問題5の選択問題は、数学科の専門科目(大学2年の後半〜大学3年)を含む内容となっています。問題がわかっている問題5−(1)については、(複素)関数論を使わなくても解けますが、内容は関数論を強く意識したものとなっています。
そのため、留数計算などの出題の可能性もあります。(他にも簡単なルベーグ積分の問題等も)


以上のような状況あるので、数学的に厳密な証明を心がけられたほうがよいと考えます。


なお、問題5ですが、
f(z)=\frac{a\overline{z}+b}{\overline{b}\overline{z}+\overline{a}}
ではないかと考えられます。(分母のb複素共役
(2010/2/1 20:38 JST actuary2さんに分母のb複素共役であることの確認がとれました。)
以下その前提で解いています。


その他の注意については
http://d.hatena.ne.jp/actuary_math/20100114
のそれと同じです。


問題1
(1)
x_{n+1}=a \cdot x_n+b \cdot y_n
y_{n+1}=c \cdot x_n+d \cdot y_n
を合計して
x_{n+1}+y_{n+1}=(a+c) \cdot x_n+(b+d) \cdot y_n
=x_n+y_n
(∵a+c=b+d=1

x_1+y_1=1
だったので、
x_n+y_n=1…(証終)


(2)
(1)より、
x_{n+1}=a \cdot x_n+b (1-x_n)
=(a-b)x_n+b

(i)a-b=1のとき、
x_{n+1}=x_n+b
と等差数列になり、
x_n=b(n-1)+x_1

(ii)a-b \ne 1のとき、
特性方程式t=(a-b)t+bの解は
t=\frac{b}{1-a+b}

x_{n+1}-\frac{b}{1-a+b}=(a-b)\left(x_n-\frac{b}{1-a+b}\right)
これより、
x_n=\frac{b}{1-a+b}+\left(x_1-\frac{b}{1-a+b}\right)(a-b)^{n-1}…(答)

(3)
(i)
[tex:-1 1]
かつ
x_1 > \frac{b}{1-a+b}
のとき、
\lim_{n \to \infty}\left(x_1-\frac{b}{1-a+b}\right)(a-b)^{n-1}=\infty
なので、
\lim_{n \to \infty}x_n=\infty

(v)
a-b > 1
かつ
x_1 < \frac{b}{1-a+b}
のとき、
\lim_{n \to \infty}\left(x_1-\frac{b}{1-a+b}\right)(a-b)^{n-1}=-\infty
なので、
\lim_{n \to \infty}x_n=-\infty

(vi)
a-b > -1
かつ
x_1 = \frac{b}{1-a+b}
のとき、
\left(x_1-\frac{b}{1-a+b}\right)(a-b)^{n-1}=0
なので、
\lim_{n \to \infty}x_n=\frac{b}{1-a+b}=x_1

(vii)
a-b > 1かつb>0のとき
\lim_{n \to \infty}x_n=\lim_{n \to \infty}\{b(n-1)+x_1\}=\infty

(viii)
a-b > 1かつb<0のとき
\lim_{n \to \infty}x_n=\lim_{n \to \infty}\{b(n-1)+x_1\}=-\infty

(ix)
a-b > 1かつb=0のとき
\lim_{n \to \infty}x_n=\lim_{n \to \infty}(x_1)=x_1


問題2
赤球の個数をNとしN=i \, (i=0,1,2)の確率をp_iとする。
p_0=\frac{{}_7C_2}{{}_10C_2}=\frac{7}{15}
p_2=\frac{{}_3C_2}{{}_10C_2}=\frac{1}{15}

p_1=1-\frac{7}{15}-\frac{1}{15}=\frac{7}{15}
(なお、
p_1=\frac{{}_7C_1 \cdot {}_3C_1}{{}_10C_2}
としてもよい)


期待値E(N)は、
E(N)=0 \cdot p_0+1 \cdot p_1+2 \cdot p_2=\frac{3}{5}
また
E(N^2)=0 \cdot p_0+1 \cdot p_1+4 \cdot p_2=\frac{11}{15}
なので
分散V(N)は、
V(N)=E(N^2)-\{E(N)\}^2=\frac{11}{15}-\frac{9}{25}=\frac{28}{75}

標準偏差\sigma(N)は、
\sigma(N)=\sqrt{V(N)}=\frac{2\sqrt{7}}{5\sqrt{3}}=\frac{2\sqrt{21}}{15}


問題4
(1)
\int_n^{n+1}\frac{dx}{x^s}
=\left[ - \frac{1}{(s-1)x^{s-1}} \right]_n^{n+1}
=\frac{1}{s-1}\left\{ \frac{1}{n^{s-1}}- \frac{1}{(n+1)^{s-1}} \right\}…(答)

(2)
a_m=\sum_{n=1}^m \frac{1}{n^s}
とする。
m \ge 2のとき、
a_m<1+\sum_{n=2}^m \left(\int_{n-1}^n\frac{dx}{x^s} \right)
=1+\frac{1}{s-1}\left( 1- \frac{1}{m^{s-1}} \right)
なので、\{a_m\}は上に有界
また、
\{a_m\}は明らかに単調増加数列なので、
「上に有界で単調増加する数列は収束する」という事実からa_mは収束し、題意は証明された。…(証終)

(3)
あるx_0について、f(x_0)=a>0とする。
このときf(x)が連続なので、
任意の\epsilon>0に対して、ある\delta>0が存在し、
|x-x_0|<\deltaとなる任意のxに対し、|f(x)-f(x_0)|<\epsilonとできる。
特に
\epsilon=\frac{a}{2}
とすると、
|f(x)-f(x_0)|<\frac{a}{2}ゆえ
f(x)>\frac{a}{2}
とできる。
g(x)=\left\{\begin{array}{ll} \frac{a}{2} && (|x-x_0|<\delta) \\ 0 && (otherwise) \end{array}\right.
とすると、
すべてのxf(x) \le g(x)なので、
\int_{-\infty}^{\infty}f(x)dx \ge \int_{-\infty}^{\infty}g(x)dx =\frac{a\delta}{2}>0
となり、
\int_{-\infty}^{\infty}f(x)dx=0
に矛盾。
∴すべてのxf(x)=0


問題5
(1)
(i)
|f(0)|^2=\left|\frac{b}{\overline{a}}\right|^2
=\frac{|b|^2}{|a|^2}
=\frac{|b|^2}{1+|b|^2}
<1

|f(0)|<1であり、f(0) \subseteq D…(証終)
(ii)
|z|=1のとき、
|a\overline{z}+b|
=|a\overline{z}+b||z|
=|a\overline{z}z+bz|
=|a+bz|
=|\overline{a+bz}|
=|\overline{b}\overline{z}+\overline{a}|…(証終)

(iii)
(解法1)
g(z)=\overline{z}とおくと、
gD \to Dの全単射かつ\partial D \to \partial Dの全単射…(a)
また、
h(z)=\frac{az+b}{\overline{b}z+\overline{a}}
f=h \circ g
であり、
f^{-1}=g \circ h^{-1}
である。
さて、|z|<1を満たすあるzに対して、|f(z)|<1でないとする。
このとき、|f(z)|=1|f(z)|>1のいずれかである。
|f(z)|>1のとき
k(t)=|f(tz)| \, (0 \le t \le 1)は、tの連続関数で、(i)よりk(0)<1なので、中間値の定理によりある[tex:t \,(0k(t)=1*1
次に、
h(z)=\frac{az+b}{\overline{b}z+\overline{a}}
とすると、
f=h \circ g

\overline{b}z+\overline{a}=0 となるのは、
z=-\frac{\overline{a}}{\overline{b}}
のときで、
|a|>|b|
のため、
\left|-\frac{\overline{a}}{\overline{b}}\right|>1
つまり、D \cup \partial D\overline{b}z+\overline{a}が0になることはないので、D \cup \partial Dh(z)は正則関数。

「最大(絶対)値の原理」(例えば、
http://ja.wikipedia.org/wiki/%E6%9C%80%E5%A4%A7%E7%B5%B6%E5%AF%BE%E5%80%A4%E3%81%AE%E5%8E%9F%E7%90%86
等参照)
により、
D \cup \partial D|h(z)|が最大値をとるのは、\partial D上。
ところが、その最大値は(ii)により、1であることがわかる。

|h(z)|<1 \, (in \, D)
このことと(a)より、
|f(z)|<1 \, (in \, D)…(証終)


(解法3)
次のようにすれば、関数論なしで(i)〜(iii)証明可能。
|a\overline{z}+b|^2
=(a\overline{z}+b)\overline{(a\overline{z}+b)}
=(a\overline{z}+b)(\overline{a}z+\overline{b})
=|a|^2|z|^2+|b|^2+\overline{a}bz+a\overline{b}\overline{z}
同様に
|\overline{b}\overline{z}+\overline{a}|^2
=|a|^2+|b|^2|z|^2+\overline{a}bz+a\overline{b}\overline{z}

|b\overline{z}+\overline{a}|^2-|a\overline{z}+b|^2
=|a|^2(1-|z|^2)+|b|^2(|z|^2-1)+(\overline{a}b-\overline{a}b)z+(a\overline{b}-a\overline{b})\overline{z}
=(|a|^2-|b|^2)(1-|z|^2)
=1-|z|^2
この値は|z|=1のとき0で、|z|<1のとき正である。
つまり、
|z|=1のとき、|a\overline{z}+b|=|\overline{b}\overline{z}+\overline{a}|
|z|<1のとき、|a\overline{z}+b|<|\overline{b}\overline{z}+\overline{a}|となるので、題意は証明された。…(証終)

以上

*1:gは正則でないことに注意!